Đến nội dung

the man nội dung

Có 595 mục bởi the man (Tìm giới hạn từ 25-05-2020)



Sắp theo                Sắp xếp  

#557840 Topic về Bất đẳng thức, cực trị THCS

Đã gửi bởi the man on 04-05-2015 - 19:31 trong Bất đẳng thức và cực trị

Cho $x,y,z>0$ thỏa mãn : $x+y+z=2015$.

Chứng minh rằng:

$\sum \frac{2015x-x^2}{yz}+6\geq 2\sqrt{2}.\sum \sqrt{\frac{2015-x}{x}}$

$VT=\sum \frac{(x+y+z)x-x^2}{yz}+6=\sum \left ( \frac{x}{y}+\frac{x}{z} \right )+6=t+6$  

    Với $t=\sum \left ( \frac{x}{y}+\frac{x}{z} \right )$

$VP=2\sqrt{2}.\sum \sqrt{\frac{y+z}{x}}\leq 2\sqrt{2}.\sqrt{3(\sum \frac{y+z}{x})}=2\sqrt{6t}$

Như vậy ta cần chứng minh

   $t+6\geq 2\sqrt{6t}$   (điều này luôn đúng với bất đẳng thức cô-si)




#557846 Topic về Bất đẳng thức, cực trị THCS

Đã gửi bởi the man on 04-05-2015 - 19:38 trong Bất đẳng thức và cực trị

Giúp mình với 
Với a,b,c >0, chứng minh rằng
$\frac{1}{a^{3}} + \frac{1}{b^{3}} + \frac{1}{c^{3}} \geq 9 (\frac{1}{a+2b}+\frac{1}{b+2c}+\frac{1}{c+2a})-6$

$\sum \frac{1}{a^3}+6=\sum \left ( \frac{1}{a^3}+1+1 \right )\geq \sum \frac{3}{a}=\sum \left ( \frac{1}{a}+\frac{1}{b}+\frac{1}{b} \right )\geq \sum \frac{9}{a+2b}\Rightarrow đpcm$




#557922 Topic về Bất đẳng thức, cực trị THCS

Đã gửi bởi the man on 05-05-2015 - 12:19 trong Bất đẳng thức và cực trị

Các bạn giúp mình bài này nhé

Cho $x$, $y$ $>$$0$ thỏa mãn $x$+ $xy$+ $y$ $=$ $8$

Tìm GTNN của BT: P=$x^{3}$+$y^{3}$+ $x^{2}$+ $y^{2}$+ $5(x+y)$+ $\frac{1}{x}$+ $\frac{1}{y}$

$9=x+y+xy+1=(x+1)(y+1)\leq \left ( \frac{x+y+2}{2} \right )^2\Rightarrow (x+y+2)^2\geq 36\Rightarrow x+y\geq 4$

$P+40=(x^3+8+8)+(y^3+8+8)+(x^2+4)+(y^2+4)+\frac{19}{4}(x+y)+\frac{1}{4}(x+y)+\frac{4}{x+y}\geq 12x+12y+4x+4y+\frac{19}{4}(x+y)+2\geq 12.4+4.4+\frac{19}{4}.4+2=85\Rightarrow P\geq 45$

$minP=45 \Leftrightarrow x=y=2$




#558017 $\boxed{\text{Chuyên Đề}}$ Bất đẳng thức - Cực trị

Đã gửi bởi the man on 05-05-2015 - 22:37 trong Bất đẳng thức và cực trị

Sai rồi bạn tử các phân thức có là bình phương đâu mà Cauchy Schwartz

Bạn ấy làm tắt chút thôi

Bạn nhân cả tử và mẫu của mỗi phân thức ở VT lần lượt với $a,b,c$ là có bình phương ngay




#559040 $\boxed{\text{Chuyên Đề}}$ Bất đẳng thức - Cực trị

Đã gửi bởi the man on 13-05-2015 - 06:45 trong Bất đẳng thức và cực trị

Cho a, b, c > 0 và $a^{2}+b^{2}+c^{2}=3$. Tìm GTNN của $P=\frac{ab^{2}+bc^{2}+ca^{2}}{\left ( ab+bc+ca \right )^{2}}$

Dễ thấy $a+b+c\leq \sqrt{3(a^2+b^2+c^2)}=3$

Theo Cauchy-Schwarz 

$(a+b+c)(ab^2+bc^2+ca^2)\geq (ab+bc+ca)^2$

 $\Rightarrow \frac{ab^2+bc^2+ca^2}{(ab+bc+ca)^2}\geq \frac{1}{a+b+c}\geq \frac{1}{3}$




#554886 Topic tổng hợp các bài toán về phương trình nghiệm nguyên.

Đã gửi bởi the man on 18-04-2015 - 19:42 trong Số học

 

2.Tìm nghiệm nguyên không âm: $x^{2}= y^{2}+\sqrt{y+1}$

 

Đặt $\sqrt{y+1}=a(a\geq 1)\Rightarrow y=a^2-1$

Thế và phương trình ta có 

               $x^2=(a^2-1)^2+a>(a^2-1)^2$

Mặt khác $x^2=a^4-(2a+1)(a-1)\leq (a^2)^2$ (do $a\geq1$)

   $\Rightarrow (a^2)^2\geq x^2>(a^2-1)^2\Rightarrow x=a^2$

   $\Rightarrow a=1\Rightarrow y=0\Rightarrow x=1$




#569410 Topic tổng hợp một số bất đẳng thức trong kì thi MO các nước

Đã gửi bởi the man on 02-07-2015 - 09:44 trong Bất đẳng thức - Cực trị

Bài 165 (Mediterranean Mathematical Competition 2009).Chứng minh rằng với mọi  $a,b,c $ dương ta luôn có:

$$\sum \frac{ab}{a^2+ab+b^2}\leq \frac{a}{2a+b}+\frac{b}{2b+c}+\frac{c}{2c+a}$$




#569421 Topic tổng hợp một số bất đẳng thức trong kì thi MO các nước

Đã gửi bởi the man on 02-07-2015 - 11:01 trong Bất đẳng thức - Cực trị

Bài 153: (Việt Nam TST 2005) 

Cho $a,b,c>0$. Chứng minh: $\left ( \frac{a}{a+b} \right )^3+\left ( \frac{b}{b+c} \right )^3+\left ( \frac{c}{c+a} \right )^3 \geq \frac{3}{8}$

$P=\left ( \frac{a}{a+b} \right )^3+\left ( \frac{b}{b+c} \right )^3+\left ( \frac{c}{c+a} \right )^3=\frac{1}{\left ( 1+\frac{b}{a} \right )^3}+\frac{1}{\left ( 1+\frac{c}{b} \right )^3}+\frac{1}{\left ( 1+\frac{a}{c} \right )^3}$

    $=\frac{1}{(1+x)^3}+\frac{1}{(1+y)^3}+\frac{1}{(1+z)^3}$       Với  $\frac{b}{a}=x,\frac{c}{b}=y,\frac{a}{c}=z\rightarrow xyz=1$

Sử dụng bất đẳng thức Am-GM:

    $\frac{1}{(1+x)^3}+\frac{1}{(1+x)^3}+\frac{1}{8}\geq \frac{3}{2}.\frac{1}{(1+x)^2}\rightarrow \frac{1}{(1+x)^3}\geq \frac{3}{4}.\frac{1}{(1+x)^2}-\frac{1}{16}$

    $\rightarrow P\geq \frac{3}{4}.\left [ \frac{1}{(1+x)^2}+\frac{1}{(1+y)^2}+ \frac{1}{(1+z)^2}\right ]-\frac{3}{16}$

Ta cần chứng minh :  

            $\frac{1}{(1+x)^2}+\frac{1}{(1+y)^2}+ \frac{1}{(1+z)^2}\geq \frac{3}{4}$    (*)

Ta có bài toán quen thuộc:

Với  $m,n,p,q$  là các số dương có tích bằng 1 thì  

            $\frac{1}{(1+m)^2}+ \frac{1}{(1+n)^2}+\frac{1}{(1+p)^2}+ \frac{1}{(1+q)^2}\geq 1$

Áp dụng vào bài toán với  $m=x,n=y,p=z,q=1$ ta có  (*)

Vậy bài toán được chứng minh xong




#569349 Topic tổng hợp một số bất đẳng thức trong kì thi MO các nước

Đã gửi bởi the man on 01-07-2015 - 19:59 trong Bất đẳng thức - Cực trị

Bài 163(Spain TST): Cho a,b,c>0: $a^2+b^2+c^2=3$. CMR:

$\frac{a}{a^2+b^3+c^2}+\frac{b}{b^2+c^3+a^2}+\frac{c}{c^2+a^3+b^2}\leq 1$

Sử dụng bất đẳng thức Holder:

    $(a^2+b^3+c^2)(a^2+b^3+c^2)(a^2+1+c^2)\geq (a^2+b^2+c^2)^3=27$

    $\rightarrow a^2+b^3+c^2\geq \frac{3\sqrt{3}}{\sqrt{a^2+c^2+1}}\rightarrow \frac{a}{a^2+b^3+c^2}\leq \frac{a\sqrt{a^2+c^2+1}}{3\sqrt{3}}$

    $\rightarrow \sum \frac{a}{a^2+b^3+c^2}\leq \frac{\sum a\sqrt{a^2+c^2+1}}{3\sqrt{3}}$

Sử dụng bất đẳng thức  Cauchy-Schwarz:

    $\sum a\sqrt{a^2+c^2+1}\leq \sqrt{(a^2+b^2+c^2)(2a^2+2b^2+2c^2+3)}=3\sqrt{3}$

Từ đó ta có điều phải chứng minh




#569423 Topic tổng hợp một số bất đẳng thức trong kì thi MO các nước

Đã gửi bởi the man on 02-07-2015 - 11:03 trong Bất đẳng thức - Cực trị

Đặt $(a;b;c)=(\frac{x}{y};\frac{y}{z};\frac{z}{x})$ , đưa bđt cần cm thành

$\sum \frac{x^{2}}{x^{3}+x^{2}+1}\leq\sum \frac{x}{2x+1}$

Hay $\sum(x-1)^{2}(x+1)x\geq 0$  (đúng)

Vậy bất đẳng thức đc cm

Dấu đẳng thức xảy ra khi $a=b=c=1$ 

Thế thì $abc=1$ à




#557207 $\boxed{\text{Chuyên Đề}}$ Phương trình vô tỉ - Hệ phương...

Đã gửi bởi the man on 01-05-2015 - 07:28 trong Phương trình, hệ phương trình và bất phương trình

Giải phương trình $\sqrt{5x-1}+\sqrt[3]{9-x}=2x^{2}+3x-1$

ĐK $x\geq \frac{1}{5}$

PT $\Leftrightarrow(\sqrt{5x-1}-2)+(\sqrt[3]{9-x}-2)=2x^2+3x-5 $

     $\Leftrightarrow \frac{5(x-1)}{\sqrt{5x-1}+2}+\frac{1-x}{\sqrt[3]{(9-x)^2}+2\sqrt[3]{9-x}+4}=(x-1)(2x+5)$

     $\Leftrightarrow (x-1).\left [ 2x+5+\frac{1}{\sqrt[3]{(9-x)^2}+2\sqrt[3]{9-x}+4}-\frac{5}{\sqrt{5x-1}+2} \right ]=0$

Do $\left\{\begin{matrix}2x+5\geq 2.\frac{1}{5}+5>\frac{5}{2}\geq \frac{5}{\sqrt{5x-1}+2} & & \\ \sqrt[3]{(9-x)^2}+2\sqrt[3]{9-x}+4\geq 0 & & \end{matrix}\right.\Rightarrow$ Biểu thức trong ngoặc vuông vô nghiệm

Như vậy $x-1=0 \Rightarrow x=1$




#558068 $\boxed{\text{Chuyên Đề}}$ Phương trình vô tỉ - Hệ phương...

Đã gửi bởi the man on 06-05-2015 - 17:32 trong Phương trình, hệ phương trình và bất phương trình

Các bài toán phương trình vô tỉ trong các đề thi HSG tỉnh

 

 

           c) $\sqrt[3]{3x^{2}-x+2001}-\sqrt[3]{3x^{2}-7x+2002}-\sqrt[3]{6x-2003}=\sqrt[3]{2002}$

 

 

Câu c sử dụng nhân liên hợp

Ngoài ra còn có thể làm thế này 

Đặt $\left\{\begin{matrix}\sqrt[3]{3x^2-x+2001}=a & & & \\ \sqrt[3]{3x^2-7x+2002}=b & & & \\ \sqrt[3]{6x-2003}=c & & &\end{matrix}\right.\Rightarrow 2002=a^3-b^3-c^3$

Pt $\Leftrightarrow a-b-c=\sqrt[3]{a^3-b^3-c^3}$

Lập phương 2 vế rồi phân tích thành nhân tử ta được $(a-b)(a-c)(b+c)=0$

Đến đây xét 3 TH là ra 




#552914 Violympic 2015

Đã gửi bởi the man on 10-04-2015 - 14:32 trong Góc giao lưu

Hixx em được có 260 điểm :((

Bạn học giỏi vậy mà đk có 260 thì chán thật




#552948 Violympic 2015

Đã gửi bởi the man on 10-04-2015 - 16:54 trong Góc giao lưu

Thế mấy bác xem 190 thì được cái gì không




#552911 Violympic 2015

Đã gửi bởi the man on 10-04-2015 - 14:19 trong Góc giao lưu

Năm ngoái 2 bé Bắc ninh 300đ nó thì sao.

Thái bình, Vĩnh phúc,... toán 290,280 mà!

lo gì thế :(

lo chứ. 

mấy bạn điểm cao nên chắc được huy chương rồi




#552909 Violympic 2015

Đã gửi bởi the man on 10-04-2015 - 14:14 trong Góc giao lưu

Trời ơi thế này thì chắc mình ko đủ điểm để đk khuyến khích mất  :(  :(  :(

Bao nhiêu đứa trên 250




#552982 Violympic 2015

Đã gửi bởi the man on 10-04-2015 - 18:53 trong Góc giao lưu

Ai biết năm ngoái bn thì đk khuyến khích ko >




#553850 Violympic 2015

Đã gửi bởi the man on 13-04-2015 - 22:40 trong Góc giao lưu

Chắc mình trượt rồi




#553215 Violympic 2015

Đã gửi bởi the man on 11-04-2015 - 17:41 trong Góc giao lưu

Mi vàng e tau ké được cái đồng đó H ơi :(

Nhục mặt thiệt :(

Về bị chưởi te tua :(

Sao bạn biết




#553005 Violympic 2015

Đã gửi bởi the man on 10-04-2015 - 19:58 trong Góc giao lưu

Các bạn thấy đề năm nay với năm rồi cái nào dễ  hơn

Mình nghĩ năm nay dễ hơn. 

Còn bài cuối thì khó ngang nhau




#553853 Violympic 2015

Đã gửi bởi the man on 13-04-2015 - 22:48 trong Góc giao lưu

http://violympic.vn/...il.aspx?ID=1279

Ấn vào cái bảng ở dưới có ghi từng lớp đó!

thế bạn biết năm ngoái có bao nhiêu ng thi ko




#554154 Violympic 2015

Đã gửi bởi the man on 15-04-2015 - 16:35 trong Góc giao lưu

Nghe nói lấy 500 hả các bạn 

ôi thôi/ lấy 600 đi , ko mình trượt




#552986 Violympic 2015

Đã gửi bởi the man on 10-04-2015 - 19:03 trong Góc giao lưu

Mình có mã bảng A thôi bạn, lấy k

T08DD071AE72

Đâu có xem được




#552906 Violympic 2015

Đã gửi bởi the man on 10-04-2015 - 14:05 trong Góc giao lưu

Các bạn ơi mình thi kém lắm. có 190 thôi  :(  :(  :(  :(

Mấy bạn siêu thật




#554981 Bất đẳng thức chuẩn bị cho kì thi THPTQG 2015-2016

Đã gửi bởi the man on 19-04-2015 - 08:21 trong Bất đẳng thức và cực trị

Cho : $x+y+z\geq 3$

Tìm min P =$\sum \frac{x^2}{yz+\sqrt{8+x^3}}$

Sử dụng Cô-si  $\sqrt{8+x^3}=\sqrt{(x+2)(x^2-2x+4)}\leq \frac{x^2-x+6}{2}\Rightarrow \frac{x^2}{yz+\sqrt{8+x^3}}\geq \frac{2x^2}{2yz+x^2-x+6}$

$\Rightarrow \sum \frac{x^2}{yz+\sqrt{8+x^3}}\geq 2.\frac{(x+y+z)^2}{x^2+y^2+z^2+2xy+2yz+2xz-x-y-z+18}$

                             $=2.\frac{(x+y+z)^2}{(x+y+z)^2-(x+y+z)+18}=2.\frac{t^2}{t^2-t+18}$      Với $t=x+y+z, t \geq3$

Nhận thấy dấu = xảy ra khi $t=3$ nên khi đó $\frac{t^2}{t^2-t+18}=\frac{3}{8}$

Ta chứng minh $\frac{t^2}{t^2-t+18}\geq \frac{3}{8}$

BĐT này tương đương với $5t^2+3t-54\geq 0$   (luôn đúng do $t \geq 3$)

$\Rightarrow Min P=\frac{3}{4}\Leftrightarrow x=y=z=1$